[skip-to-content]
\(\newcommand{\identity}{\mathrm{id}} \newcommand{\notdivide}{{\not{\mid}}} \newcommand{\notsubset}{\not\subset} \newcommand{\lcm}{\operatorname{lcm}} \newcommand{\gf}{\operatorname{GF}} \newcommand{\inn}{\operatorname{Inn}} \newcommand{\aut}{\operatorname{Aut}} \newcommand{\Hom}{\operatorname{Hom}} \newcommand{\cis}{\operatorname{cis}} \newcommand{\chr}{\operatorname{char}} \newcommand{\Null}{\operatorname{Null}} \renewcommand{\gcd}{\operatorname{mcd}} \renewcommand{\lcm}{\operatorname{mcm}} \renewcommand{\deg}{\operatorname{gr}} \newcommand{\lt}{<} \newcommand{\gt}{>} \newcommand{\amp}{&} \)

Sección17.2El Algoritmo de División

Recuerde que el algoritmo de división para enteros (Teorema 2.9) dice que si \(a\) y \(b\) son enteros con \(b \gt 0\text{,}\) entonces existen únicos enteros \(q\) y \(r\) tales que \(a = bq+r\text{,}\) con \(0 \leq r \lt b\text{.}\) Un teorema similar existe para polinomios. El algoritmo de división para polinomios tiene varias consecuencias importantes. Como su demostración es muy similar a la demostración correspondiente para los enteros, resulta conveniente revisar el Teorema 2.9 antes de seguir.

Primero demostraremos la existencia de \(q(x)\) y \(r(x)\text{.}\) Si \(f(x)\) es el polinomio cero, entonces

\begin{equation*} 0 = 0 \cdot g(x) + 0; \end{equation*}

luego, tanto \(q\) como \(r\) también son el polinomio cero. Ahora supongamos que \(f(x)\) no es polinomio cero y que \(\deg f(x) = n\) y \(\deg g(x) = m\text{.}\) Si \(m \gt n\text{,}\) entonces \(q(x) = 0\) y \(r(x) = f(x)\text{.}\) Podemos ahora suponer que \(m \leq n\) y proceder por inducción en \(n\text{.}\) Si

\begin{align*} f(x) & = a_n x^n + a_{n-1} x^{n - 1} + \cdots + a_1 x + a_0\\ g(x) & = b_m x^m + b_{m-1} x^{m - 1} + \cdots + b_1 x + b_0 \end{align*}

entonces el polinomio

\begin{equation*} f'(x) = f(x) - \frac{a_n}{b_m} x^{n - m} g(x) \end{equation*}

tiene grado menor a \(n\) o es el polinomio cero. Por la hipótesis de inducción, existens polinomios \(q'(x)\) y \(r(x)\) tales que

\begin{equation*} f'(x) = q'(x) g(x) + r(x), \end{equation*}

donde \(r(x) = 0\) o el grado de \(r(x)\) es menor al grado de \(g(x)\text{.}\) Ahora, sea

\begin{equation*} q(x) = q'(x) + \frac{a_n}{b_m} x^{n - m}. \end{equation*}

Entonces

\begin{equation*} f(x) = g(x) q(x) + r(x), \end{equation*}

con \(r(x)\) el polinomio cero o \(\deg r(x) \lt \deg g(x)\text{.}\)

Para mostrar que \(q(x)\) y \(r(x)\) son únicos, supongamos que además existen \(q_1(x)\) y \(r_1(x)\) tales que \(f(x) = g(x) q_1(x) + r_1(x)\) con \(\deg r_1(x) \lt \deg g(x)\) o \(r_1(x) = 0\text{,}\) de manera que

\begin{equation*} f(x) = g(x) q(x) + r(x) = g(x) q_1(x) + r_1(x), \end{equation*}

y

\begin{equation*} g(x) [q(x) - q_1(x) ] = r_1(x) - r(x). \end{equation*}

Si \(g(x)\) no es el polinomio cero, entonces

\begin{equation*} \deg( g(x) [q(x) - q_1(x) ] )= \deg( r_1(x) - r(x) ) \geq \deg g(x). \end{equation*}

Pero, los grados tanto de \(r(x)\) como de \(r_1(x)\) son estrictamente menores que el grado de \(g(x)\text{;}\) por lo tanto, \(r(x) = r_1(x)\) y \(q(x) = q_1(x)\text{.}\)

Ejemplo17.7

El algoritmo de división meramente formaliza la división larga de polinomios, una tarea con la que probablemente estamos familiarizados desde el colegio. Por ejemplo, supongamos que dividimos \(x^3 - x^2 + 2 x - 3\) por \(x - 2\text{.}\)

\(x^2\) \(+\) \(x\) \(+\) \(4\)
\(x\) \(-\) \(2\) \(x^3\) \(-\) \(x^2\) \(+\) \(2x\) \(-\) \(3\)
\(x^3\) \(-\) \(2x^2\)
\(x^2\) \(+\) \(2x\) \(-\) \(3\)
\(x^2\) \(-\) \(2x\)
\(4x\) \(-\) \(3\)
\(4x\) \(-\) \(8\)
\(5\)

Luego, \(x^3 - x^2 + 2 x - 3 = (x - 2) (x^2 + x + 4 ) + 5\text{.}\)

Sea \(p(x)\) un polinomio en \(F[x]\) y \(\alpha \in F\text{.}\) Decimos que \(\alpha\) es un cero o raíz de \(p(x)\) si \(p(x)\) está en el núcleo del homomorfismo de evaluación \(\phi_{\alpha}\text{.}\) Lo único que estamos dicendo realmente es que \(\alpha\) es un cero de \(p(x)\) si \(p(\alpha) = 0\text{.}\)

Supongamos que \(\alpha \in F\) y \(p( \alpha ) = 0\text{.}\) Por el algoritmo de la división, existen polinomios \(q(x)\) y \(r(x)\) tales que

\begin{equation*} p(x) = (x -\alpha) q(x) + r(x) \end{equation*}

y el grado de \(r(x)\) es menor que el grado de \(x -\alpha\text{.}\) Como el grado de \(r(x)\) es menor a 1, \(r(x) = a\) para algún \(a \in F\text{;}\) por lo tanto,

\begin{equation*} p(x) = (x -\alpha) q(x) + a. \end{equation*}

Pero

\begin{equation*} 0 = p(\alpha) = 0 \cdot q(\alpha) + a = a; \end{equation*}

Por ende, \(p(x) = (x - \alpha) q(x)\text{,}\) y \(x - \alpha\) es un factor de \(p(x)\text{.}\)

Recíprocamente, supongamos que \(x - \alpha\) es un factor de \(p(x)\text{;}\) digamos \(p(x) = (x - \alpha) q(x)\text{.}\) Entonces \(p( \alpha ) = 0 \cdot q(\alpha) = 0\text{.}\)

Procederemos por inducción sobre el grado de \(p(x)\text{.}\) Si \(\deg p(x) = 0\text{,}\) entonces \(p(x)\) es un polinomio constante y no tiene ceros. Si \(\deg p(x) = 1\text{,}\) entonces \(p(x) = ax +b\) para ciertos \(a\) y \(b\) en \(F\text{.}\) Si \(\alpha_1\) y \(\alpha_2\) so ceros de \(p(x)\text{,}\) entonces \(a\alpha_1 + b = a\alpha_2 +b\) y \(\alpha_1 = \alpha_2\text{.}\)

Ahora supongamos que \(\deg p(x) \gt 1\text{.}\) Si \(p(x)\) no tiene ceros en \(F\text{,}\) estamos listos. Por otra parte, si \(\alpha\) es un cero de \(p(x)\text{,}\) entonces \(p(x) = (x - \alpha ) q(x)\) para cierto \(q(x) \in F[x]\) por el Corolario 17.8. El grado de \(q(x)\) es \(n-1\) por la Proposición 17.4. Sea \(\beta\) algún otro cero de \(p(x)\) distinto de \(\alpha\text{.}\) Entonces \(p(\beta) = (\beta - \alpha) q(\beta) = 0\text{.}\) Como \(\alpha \neq \beta\) y \(F\) es un cuerpo, \(q(\beta ) = 0\text{.}\) Por la hipótesis de inducción, \(q(x)\) puede tener a lo sumo \(n -1\) ceros distintos en \(F\text{.}\) Por lo tanto, \(p(x)\) tiene a lo sumo \(n\) ceros distintos en \(F\text{.}\)

Sea \(F\) un cuerpo. Un polinomio mónico \(d(x)\) es un máximo común divisor de los polinomios \(p(x), q(x) \in F[x]\) si \(d(x)\) divide tanto a \(p(x)\) como a \(q(x)\text{;}\) y, si para cualquier otro polinomio \(d'(x)\) que divida tanto a \(p(x)\) como a \(q(x)\text{,}\) \(d'(x) \mid d(x)\text{.}\) Escribiremos \(d(x) = \gcd( p(x), q( x))\text{.}\) Dos polinomios \(p(x)\) y \(q(x)\) son relativamente primos si \(\gcd(p(x), q(x) ) = 1\text{.}\)

Sea \(d(x)\) el polinomio mónico de menor grado en el conjunto

\begin{equation*} S = \{ f(x) p(x) + g(x) q(x) : f(x), g(x) \in F[x] \}. \end{equation*}

Podemos escribir \(d(x) = r(x) p(x) + s(x) q(x)\) para dos polinomios \(r(x)\) y \(s(x)\) en \(F[x]\text{.}\) Debemos demostrar que \(d(x)\) divide a \(p(x)\) y a \(q(x)\text{.}\) Primero mostraremos que \(d(x)\) divide a \(p(x)\text{.}\) Por el algoritmo de división, existen polinomios \(a(x)\) y \(b(x)\) tales que \(p(x) = a(x) d(x) + b(x)\text{,}\) donde \(b(x)\) es el polinomio cero o \(\deg b(x) \lt \deg d(x)\text{.}\) Por lo tanto,

\begin{align*} b(x) & = p(x) - a(x) d(x)\\ & = p(x) - a(x)( r(x) p(x) + s(x) q(x)) \\ & = p(x) - a(x) r(x) p(x) - a(x) s(x) q(x)\\ & = p(x)( 1 - a(x) r(x) ) + q(x) ( - a(x) s(x) ) \end{align*}

es una combinación lineal de \(p(x)\) y \(q(x)\) y por lo tanto está en \(S\text{.}\) Entonces, \(b(x)\) debe ser el polinomio cero pues \(d(x)\) fue elegido de grado minimal; Concluimos que \(d(x)\) divide a \(p(x)\text{.}\) Un argumento simétrico muestra que \(d(x)\) también divide a \(q(x)\text{;}\) luego, \(d(x)\) es un divisor común de \(p(x)\) y \(q(x)\text{.}\)

Para mostrar que \(d(x)\) es un máximo común divisor de \(p(x)\) y \(q(x)\text{,}\) supongamos que \(d'(x)\) es otro divisor común de \(p(x)\) y \(q(x)\text{.}\) Mostraremos que \(d'(x) \mid d(x)\text{.}\) Como \(d'(x)\) es un divisor común de \(p(x)\) y \(q(x)\text{,}\) existen polinomios \(u(x)\) y \(v(x)\) tales que \(p(x) = u(x) d'(x)\) y \(q(x) = v(x) d'(x)\text{.}\) Por lo tanto,

\begin{align*} d(x) & = r(x) p(x) + s(x) q(x)\\ & = r(x) u(x) d'(x) + s(x) v(x) d'(x)\\ & = d'(x) [r(x) u(x) + s(x) v(x)]. \end{align*}

Como \(d'(x) \mid d(x)\text{,}\) \(d(x)\) es un máximo común divisor de \(p(x)\) y \(q(x)\text{.}\)

Finalmente, debemos mostrar que el máximo común divisor de \(p(x)\) y \(q(x)\) es único. Supongamos que \(d'(x)\) también es un máximo común divisor de \(p(x)\) y \(q(x)\text{.}\) Acabamos de mostrar que existen polinomios \(u(x)\) y \(v(x)\) en \(F[x]\) tales que \(d(x) = d'(x)[r(x) u(x) + s(x) v(x)]\text{.}\) Como

\begin{equation*} \deg d(x) = \deg d'(x) + \deg[r(x) u(x) + s(x) v(x)] \end{equation*}

y \(d(x)\) y \(d'(x)\) son ambos máximo común divisor, \(\deg d(x) = \deg d'(x)\text{.}\) Como \(d(x)\) y \(d'(x)\) son ambos polinomios mónicos del mismo grado, se debe tener que \(d(x) =~d'(x)\text{.}\)

Notemos la similaridad entre la demostración de la Proposición 17.10 y la demostración del Teorema 2.10.